Yes, $2^{2^{2^n}}$ is a good example for $p(n)$ and $q(n)$ could be $n!$.

  • MHB
  • Thread starter mathmari
  • Start date
  • Tags
    Functions
In summary, the order of the functions from the smallest to largest is: $1, n^{\frac{1}{\lg n}}, (\sqrt{2})^{\lg n}, n, 4^{\lg n}, \frac{n}{\lg n}, \lg^2 n, \ln n, \sqrt{\lg n}, e^{\log
  • #1
mathmari
Gold Member
MHB
5,049
7
Hey! :eek:

Find an order $f_1, f_2, \dots f_{30}$ of the functions that satisfies the relations $f_1=\Omega(f_2), f_2=\Omega(f_3), \dots, f_{29}=\Omega(f_{30})$$$\frac{n}{\lg n} , \ \ n^{\lg n} ,\ \ (\sqrt{2})^{\lg n}, \ \ n^2, \ \ n!, \ \ (\lg n)! ,\ \ \left( \frac{3}{2} \right)^n ,\ \ n^3 ,\ \ \lg^2 n ,\ \ \lg(n!) ,\ \ 2^{2^n}, \ \ n^{\frac{1}{\lg n}}, \ \ \ln{\ln n}, \ \ e^{\log_{10} n }, \ \ n \cdot 2^n, \ \ n^{\lg{\lg n}}, \ \ \ln n, \ \ 1 ,\ \ 2^{\lg n}, \ \ (\lg n)^{\lg n}, \ \ e^n ,\ \ 4^{\lg n}, \ \ (n+1)!, \ \ \sqrt{\lg n}, \ \ \lg{\lg{\lg n}}, \ \ 2^{\sqrt{2 \lg n}}, \ \ n, \ \ 2^n ,\ \ n \lg n, \ \ 2^{2^{n+1}}$$
How can I find the order of the functions?? (Wondering) Do I have to find the limits of all the functions pairwise? Or is there an easier and faster way?? (Wondering)

I have done the following: The functions are:

$$\frac{n}{\lg n} , \ \ n^{\lg n} ,\ \ (\sqrt{2})^{\lg n}=\sqrt{n}, \ \ n^2, \ \ n!, \ \ (\lg n)! ,\ \ \left( \frac{3}{2} \right)^n ,\ \ n^3 ,\ \ \lg^2 n ,\ \ \lg(n!) ,\ \ 2^{2^n}, \ \ n^{\frac{1}{\lg n}}=2, \ \ \ln{\ln n}, \ \ e^{\log_{10} n }, \ \ n \cdot 2^n, \ \ n^{\lg{\lg n}}, \ \ \ln n, \ \ 1 ,\ \ 2^{\lg n}, \ \ (\lg n)^{\lg n}, \ \ e^n ,\ \ 4^{\lg n}=n^2, \ \ (n+1)!, \ \ \sqrt{\lg n}, \ \ \lg{\lg{\lg n}}, \ \ 2^{\sqrt{2 \lg n}}, \ \ n, \ \ 2^n ,\ \ n \lg n, \ \ 2^{2^{n+1}}$$We know that $$\log^a n < n^b < c^n, \forall a, b>0, \forall c>1$$

Is the following classification correct?? (Wondering)

Constants: $1, n^{\frac{1}{\lg n}}$

Logarithms: $\lg{\lg{\lg n}}, \lg^2 n, \lg(n!), \sqrt{\lg n}, (\lg n)^{\lg n}, \ln{\ln n}, \ln n$

Polynomials: $(\sqrt{2})^{\lg n}, n^{\lg n}, n^2, n^3, n,e^{\log_{10} n } ,4^{\lg n}, \frac{n}{\lg n}$

Exponentials: $n!, (\lg n)!, \left( \frac{3}{2} \right)^n, 2^{2^n}, n \cdot 2^n ,2^{\lg n}, e^n, (n+1)!, 2^{\lg n}, 2^n, n \lg n, 2^{\sqrt{2 \lg n}}, 2^{2^{n+1}}$
 
Physics news on Phys.org
  • #2
Hey! ;)

mathmari said:
Do I have to find the limits of all the functions pairwise? Or is there an easier and faster way?? (Wondering)

I think so, although I'd skip the obvious ones.

For $f_1(n)=\Omega(f_2(n))$, we need that $f_1(n) \ge k\cdot f_2(n)$ for some constant $k$ and any sufficiently large $n$.
That is under the assumption that all functions are positive which they are.
It is true if
$$\lim_{n\to \infty} \frac{f_2(n)}{f_1(n)}$$
exists. (Nerd)

So in case of doubt we need to verify if the limit exists for 2 successive functions. (Thinking)
Is the following classification correct?? (Wondering)

Suppose we take $f_1=n^2$ and $f_2=n^3$, then we need that:
$$n^2=\Omega(n^3)$$
That is, $n^2$ is bounded from below by $n^3$.
Is that true? (Wondering)
 
  • #3
I like Serena said:
Suppose we take $f_1=n^2$ and $f_2=n^3$, then we need that:
$$n^2=\Omega(n^3)$$
That is, $n^2$ is bounded from below by $n^3$.
Is that true? (Wondering)

No, it is not true... It is $$n^3=\Omega(n^2)$$

I haven't find the right order yet...

Since it stands that $$\log^a n <n^b <c^n, \forall a, b>0 \text{ and } \forall c>1 \tag 1$$

I want to know which of the functions are the logarithmic functions , which are the polynomial functions and which are the exponential functions so that using the inequality $(1)$ I will not have to find the limits of all the functions pairwise.

Or can we not say for example that $n^{\lg \lg n}$ belongs to the polynomials, because it is a composition of a polynomial and a logarithmic function?? (Wondering)
 
  • #4
mathmari said:
No, it is not true... It is $$n^3=\Omega(n^2)$$

I haven't find the right order yet...

Ah, okay. (Blush)
Since it stands that $$\log^a n <n^b <c^n, \forall a, b>0 \text{ and } \forall c>1 \tag 1$$

I want to know which of the functions are the logarithmic functions , which are the polynomial functions and which are the exponential functions so that using the inequality $(1)$ I will not have to find the limits of all the functions pairwise.

Or can we not say for example that $n^{\lg \lg n}$ belongs to the polynomials, because it is a composition of a polynomial and a logarithmic function?? (Wondering)

I'm afraid that the functions you've classified as polynomials, are not in fact polynomials. That is simply because thay are not of the form $a_0+a_1 n +a_2 n^2 +...$.

However, we can say for instance: $n^{\lg \lg n} > n^k$ for any constant $k>0$ and any sufficiently large $n$.
That is, $n^{\lg \lg n}$ is asymptotically larger than any polynomial.
So we can classify it beyond the polynomials.
We still have to figure out where it is in relation to the exponentials though. (Wasntme)
 
  • #5
Can we formulate it as followed?? (Wondering)

$1$ and $n^{\frac{1}{\lg n}}=2$ are both constants so they are at the same equivalence class, therefore $1 \sim n^{\frac{1}{\lg n}}$

It stands that:

- $n^3 > n^2 \sim 4^{\lg n}=n^2 > n \sim 2^{\lg n}=n > (\sqrt{2})^{\lg n}=\sqrt{n}=n^{\frac{1}{2}} > e^{\log_{10} n }=n^{\frac{1}{\ln 10}} \Rightarrow n^3 > n^2 \sim 4^{\lg n} > n \sim 2^{\lg n} > (\sqrt{2})^{\lg n} > e^{\log_{10} n }$

- $e^n > 2^n > \left( \frac{3}{2} \right)^n$

- $2^{2^{n+1}}=2^{2^n 2}=4^{2^n} > 2^{2^n} \Rightarrow 2^{2^{n+1}} > 2^{2^n}$

- $\lg^2 n > \ln n > \sqrt{\lg n}=\lg^{\frac{1}{2}} n \Rightarrow \lg^2 n > \ln n > \sqrt{\lg n}$

$\lim_{n \rightarrow +\infty} \frac{2^{2^n}}{e^n}= \dots = \infty \Rightarrow 2^{2^n} > e^n$

Since it stands that $$c^n > n^b> \log^a n, \forall a, b >0 \text{ and } c>1$$ we have that:

$$2^{2^{n+1}} > 2^{2^n}> e^n > 2^n > \left( \frac{3}{2} \right)^n> n^3 > n^2 \sim 4^{\lg n} > n \sim 2^{\lg n} > (\sqrt{2})^{\lg n} > e^{\log_{10} n }> \lg^2 n > \ln n > \sqrt{\lg n} > 1 \sim n^{\frac{1}{\lg n}}$$

$\lim_{n \rightarrow +\infty} \frac{n}{\frac{n}{\lg n}}=\lim_{n \rightarrow +\infty} \lg n=\infty \Rightarrow n > \frac{n}{\lg n}$

$\lim_{n \rightarrow +\infty} \frac{\frac{n}{\lg n}}{(\sqrt{2})^{\lg n}}= \dots = \infty \Rightarrow \frac{n}{\lg n} > (\sqrt{2})^{\lg n}$

So, $$2^{2^{n+1}} > 2^{2^n}> e^n > 2^n > \left( \frac{3}{2} \right)^n> n^3 > n^2 \sim 4^{\lg n} > n \sim 2^{\lg n} > \frac{n}{\lg n} > (\sqrt{2})^{\lg n} > e^{\log_{10} n }> \lg^2 n > \ln n > \sqrt{\lg n} > 1 \sim n^{\frac{1}{\lg n}}$$

$\lim_{n \rightarrow +\infty} \frac{n^{\lg n} }{}=\lim_{n \rightarrow +\infty} n^{\lg n -3}=\infty \Rightarrow n^{\lg n} > n^3$

$\lim_{n \rightarrow +\infty} \frac{n^{\lg n}}{\left( \frac{3}{2} \right)^n}= \dots =0 \Rightarrow \left( \frac{3}{2} \right)^n > n^{\lg n}$

So, $$2^{2^{n+1}} > 2^{2^n}> e^n > 2^n > \left( \frac{3}{2} \right)^n> n^{\lg n}> n^3 > n^2 \sim 4^{\lg n} > n \sim 2^{\lg n} > \frac{n}{\lg n} > (\sqrt{2})^{\lg n} > e^{\log_{10} n }> \lg^2 n > \ln n > \sqrt{\lg n} > 1 \sim n^{\frac{1}{\lg n}}$$

$\lim_{n \rightarrow +\infty} \frac{n!}{2^{2^{n+1}}} = \dots =0 \Rightarrow 2^{2^{n+1}} > n!$

$\lim_{n \rightarrow +\infty} \frac{n!}{2^{2^n}}= \dots =0 \Rightarrow 2^{2^n} > n!$

$\lim_{n \rightarrow +\infty} \frac{n!}{e^n}= \dots =\infty \Rightarrow n!> e^n$

So, $$2^{2^{n+1}} > 2^{2^n}> n! > e^n > 2^n > \left( \frac{3}{2} \right)^n> n^{\lg n}> n^3 > n^2 \sim 4^{\lg n} > n \sim 2^{\lg n} > \frac{n}{\lg n} > (\sqrt{2})^{\lg n} > e^{\log_{10} n }> \lg^2 n > \ln n > \sqrt{\lg n} > 1 \sim n^{\frac{1}{\lg n}}$$

$\lim_{n \rightarrow +\infty} \frac{(\lg n)!}{n!}= \dots =0 \Rightarrow n! > (\lg n)!$

$\lim_{n \rightarrow +\infty} \frac{(\lg n)!}{e^n}= \dots =0 \Rightarrow e^n > (\lg n)!$

$\lim_{n \rightarrow +\infty} \frac{(\lg n)!}{2^n}= \dots =0 \Rightarrow 2^n > (\lg n)!$

$\lim_{n \rightarrow +\infty} \frac{(\lg n)!}{\left( \frac{3}{2} \right)^n}= \dots =0 \Rightarrow \left( \frac{3}{2} \right)^n > (\lg n)!$

$\lim_{n \rightarrow +\infty} \frac{(\lg n)!}{n^{\lg n}}= \dots =0 \Rightarrow n^{\lg n} > (\lg n)!$

$\lim_{n \rightarrow +\infty} \frac{(\lg n)!}{n^3}= \dots =\infty \Rightarrow (\lg n)! > n^3$

So, $$2^{2^{n+1}} > 2^{2^n}> n! > e^n > 2^n > \left( \frac{3}{2} \right)^n> n^{\lg n}> (\lg n)! > n^3 > n^2 \sim 4^{\lg n} > n \sim 2^{\lg n} > \frac{n}{\lg n} > (\sqrt{2})^{\lg n} > e^{\log_{10} n }> \lg^2 n > \ln n > \sqrt{\lg n} > 1 \sim n^{\frac{1}{\lg n}}$$

$\lim_{n \rightarrow +\infty} \frac{\lg(n!)}{\lg^2 n}= \dots =\infty \Rightarrow \lg(n!) > \lg^2 n$

$\lim_{n \rightarrow +\infty} \frac{\lg(n!)}{e^{\log_{10} n }}= \dots = \infty \Rightarrow \lg(n!) > e^{\log_{10} n }$

$\lim_{n \rightarrow +\infty} \frac{\lg(n!)}{(\sqrt{2})^{\lg n}}= \dots =\infty \Rightarrow \lg(n!) > (\sqrt{2})^{\lg n}$

$\lim_{n \rightarrow +\infty} \frac{\lg(n!)}{\frac{n}{\lg n} }= \dots =\infty \Rightarrow \lg(n!) > \frac{n}{\lg n} $

$\lim_{n \rightarrow +\infty} \frac{\lg(n!)}{n}= \dots =\infty \Rightarrow \lg(n!) > n$

$\lim_{n \rightarrow +\infty} \frac{\lg(n!)}{n^2 }= \dots =0 \Rightarrow n^2 > \lg(n!)$

So, $$2^{2^{n+1}} > 2^{2^n}> n! > e^n > 2^n > \left( \frac{3}{2} \right)^n> n^{\lg n}> (\lg n)! > n^3 > n^2 \sim 4^{\lg n} > \lg(n!) > n \sim 2^{\lg n} > \frac{n}{\lg n} > (\sqrt{2})^{\lg n} > e^{\log_{10} n }> \lg^2 n > \ln n > \sqrt{\lg n} > 1 \sim n^{\frac{1}{\lg n}}$$

$\lim_{n \rightarrow +\infty} \frac{\ln{\ln n}}{\ln n}= \dots =0 \Rightarrow \ln n > \ln{\ln n}$

$\lim_{ n \rightarrow +\infty} \frac{\ln{\ln n}}{\sqrt{\lg n}}= \dots =0 \Rightarrow \sqrt{\lg n} > \ln{\ln n}$

So, $$2^{2^{n+1}} > 2^{2^n}> n! > e^n > 2^n > \left( \frac{3}{2} \right)^n> n^{\lg n}> (\lg n)! > n^3 > n^2 \sim 4^{\lg n} > \lg(n!) > n \sim 2^{\lg n} > \frac{n}{\lg n} > (\sqrt{2})^{\lg n} > e^{\log_{10} n }> \lg^2 n > \ln n > \sqrt{\lg n} > \ln{\ln n} > 1 \sim n^{\frac{1}{\lg n}}$$

$\lim_{n \rightarrow +\infty} \frac{n \cdot 2^n}{2^n} \lim_{n \rightarrow +\infty} n=\infty \Rightarrow n \cdot 2^n > 2^n$

$\lim_{ n \rightarrow +\infty} \frac{n \cdot 2^n}{e^n}= \dots =0 \Rightarrow e^n > n \cdot 2^n$

So, $$2^{2^{n+1}} > 2^{2^n}> n! > e^n > n \cdot 2^n > 2^n > \left( \frac{3}{2} \right)^n> n^{\lg n}> (\lg n)! > n^3 > n^2 \sim 4^{\lg n} > \lg(n!) > n \sim 2^{\lg n} > \frac{n}{\lg n} > (\sqrt{2})^{\lg n} > e^{\log_{10} n }> \lg^2 n > \ln n > \sqrt{\lg n} > \ln{\ln n} > 1 \sim n^{\frac{1}{\lg n}}$$

$\lim_{ n \rightarrow +\infty} \frac{n^{\lg{\lg n}}}{n^{\lg n}}= \dots =0 \Rightarrow n^{\lg n} > n^{\lg{\lg n}}$

$\lim_{n \rightarrow +\infty} \frac{n^{\lg{\lg n}}}{(\lg n)!}= \dots =\infty \Rightarrow n^{\lg{\lg n}} > (\lg n)!$

So, $$2^{2^{n+1}} > 2^{2^n}> n! > e^n > n \cdot 2^n > 2^n > \left( \frac{3}{2} \right)^n> n^{\lg n}> n^{\lg{\lg n}} > (\lg n)! > n^3 > n^2 \sim 4^{\lg n} > \lg(n!) > n \sim 2^{\lg n} > \frac{n}{\lg n} > (\sqrt{2})^{\lg n} > e^{\log_{10} n }> \lg^2 n > \ln n > \sqrt{\lg n} > \ln{\ln n} > 1 \sim n^{\frac{1}{\lg n}}$$

$\lim_{n \rightarrow +\infty} \frac{(\lg n)^{\lg n}}{(\lg n)!}= \dots =\infty \Rightarrow (\lg n)^{\lg n} > (\lg n)!$

$\lim_{n \rightarrow +\infty} \frac{(\lg n)^{\lg n}}{n^{\lg{\lg n}}}= \dots =1 \Rightarrow (\lg n)^{\lg n} \sim n^{\lg{\lg n}}$

So, $$2^{2^{n+1}} > 2^{2^n}> n! > e^n > n \cdot 2^n > 2^n > \left( \frac{3}{2} \right)^n> n^{\lg n}> n^{\lg{\lg n}} \sim (\lg n)^{\lg n}> (\lg n)! > n^3 > n^2 \sim 4^{\lg n} > \lg(n!) > n \sim 2^{\lg n} > \frac{n}{\lg n} > (\sqrt{2})^{\lg n} > e^{\log_{10} n }> \lg^2 n > \ln n > \sqrt{\lg n} > \ln{\ln n} > 1 \sim n^{\frac{1}{\lg n}}$$

$\lim_{n \rightarrow +\infty} \frac{(n+1)!}{n!}=\lim_{n \rightarrow +\infty} \frac{(n!(n+1)}{n!}=\lim_{n \rightarrow +\infty} (n+1)=\infty \Rightarrow (n+1)! > n!$

$\lim_{n \rightarrow +\infty} \frac{(n+1)!}{2^{2^n}}= \dots =0 \Rightarrow 2^{2^n} > (n+1)!$

So, $$2^{2^{n+1}} > 2^{2^n}> (n+1)! > n! > e^n > n \cdot 2^n > 2^n > \left( \frac{3}{2} \right)^n> n^{\lg n}> n^{\lg{\lg n}} \sim (\lg n)^{\lg n}> (\lg n)! > n^3 > n^2 \sim 4^{\lg n} > \lg(n!) > n \sim 2^{\lg n} > \frac{n}{\lg n} > (\sqrt{2})^{\lg n} > e^{\log_{10} n }> \lg^2 n > \ln n > \sqrt{\lg n} > \ln{\ln n} > 1 \sim n^{\frac{1}{\lg n}}$$

$\lim_{n \rightarrow +\infty} \frac{\lg{\lg{\lg n}}}{\ln{\ln n} }= \dots =0 \Rightarrow \ln{\ln n} > \lg{\lg{\lg n}}$

So, $$2^{2^{n+1}} > 2^{2^n}> (n+1)! > n! > e^n > n \cdot 2^n > 2^n > \left( \frac{3}{2} \right)^n> n^{\lg n}> n^{\lg{\lg n}} \sim (\lg n)^{\lg n}> (\lg n)! > n^3 > n^2 \sim 4^{\lg n} > \lg(n!) > n \sim 2^{\lg n} > \frac{n}{\lg n} > (\sqrt{2})^{\lg n} > e^{\log_{10} n }> \lg^2 n > \ln n > \sqrt{\lg n} > \ln{\ln n} > \lg{\lg{\lg n}} > 1 \sim n^{\frac{1}{\lg n}}$$

$\lim_{n \rightarrow +\infty} \frac{2^{\sqrt{2 \lg n}}}{(\sqrt{2})^{\lg n}}= \dots =0 \Rightarrow (\sqrt{2})^{\lg n} > 2^{\sqrt{2 \lg n}}$

$\lim_{n \rightarrow +\infty} \frac{2^{\sqrt{2 \lg n}}}{e^{\log_{10} n }}= \dots =0 \Rightarrow e^{\log_{10} n } > 2^{\sqrt{2 \lg n}}$

$\lim_{n \rightarrow +\infty} \frac{2^{\sqrt{2 \lg n}}}{\lg^2 n}= \dots =\infty \Rightarrow 2^{\sqrt{2 \lg n}} > \lg^2 n$

So, $$2^{2^{n+1}} > 2^{2^n}> (n+1)! > n! > e^n > n \cdot 2^n > 2^n > \left( \frac{3}{2} \right)^n> n^{\lg n}> n^{\lg{\lg n}} \sim (\lg n)^{\lg n}> (\lg n)! > n^3 > n^2 \sim 4^{\lg n} > \lg(n!) > n \sim 2^{\lg n} > \frac{n}{\lg n} > (\sqrt{2})^{\lg n} > e^{\log_{10} n }> 2^{\sqrt{2 \lg n}} > \lg^2 n > \ln n > \sqrt{\lg n} > \ln{\ln n} > \lg{\lg{\lg n}} > 1 \sim n^{\frac{1}{\lg n}}$$

$\lim_{n \rightarrow +\infty} \frac{n \lg n}{\lg(n!)}= \dots =1 \Rightarrow n \lg n \sim \lg(n!)$

So, $$2^{2^{n+1}} > 2^{2^n}> (n+1)! > n! > e^n > n \cdot 2^n > 2^n > \left( \frac{3}{2} \right)^n> n^{\lg n}> n^{\lg{\lg n}} \sim (\lg n)^{\lg n}> (\lg n)! > n^3 > n^2 \sim 4^{\lg n} > \lg(n!) \sim n \lg n > n \sim 2^{\lg n} > \frac{n}{\lg n} > (\sqrt{2})^{\lg n} > e^{\log_{10} n }> 2^{\sqrt{2 \lg n}} > \lg^2 n > \ln n > \sqrt{\lg n} > \ln{\ln n} > \lg{\lg{\lg n}} > 1 \sim n^{\frac{1}{\lg n}}$$

Is it correct?? (Wondering) Could I improve something at the way I justified it?? (Wondering)
 
Last edited by a moderator:
  • #6
Also, I have to find a non-negative function $g(n)$ such that for all the above function $f_i(n)$, $g(n)$ is nor $O(f_i(n))$ neither $\Omega (f_i(n))$.

Could you give me some hints how I could find such a function?? (Wondering)
 
  • #7
mathmari said:
So, $$2^{2^{n+1}} > 2^{2^n}> (n+1)! > n! > e^n > n \cdot 2^n > 2^n > \left( \frac{3}{2} \right)^n> n^{\lg n}> n^{\lg{\lg n}} \sim (\lg n)^{\lg n}> (\lg n)! > n^3 > n^2 \sim 4^{\lg n} > \lg(n!) \sim n \lg n > n \sim 2^{\lg n} > \frac{n}{\lg n} > (\sqrt{2})^{\lg n} > e^{\log_{10} n }> 2^{\sqrt{2 \lg n}} > \lg^2 n > \ln n > \sqrt{\lg n} > \ln{\ln n} > \lg{\lg{\lg n}} > 1 \sim n^{\frac{1}{\lg n}}$$

Is it correct?? (Wondering) Could I improve something at the way I justified it?? (Wondering)

Looks good to me. (Nod)

I selected the first 6 and put them in this plot.
It confirms that those are in the correct order. (Happy)

For a justification, I think you only need to give the limits that prove that your order is the correct one. No need to do so many more. It would be nice if some of the actual calculation of the non-trivial ones were shown. (Nerd)
mathmari said:
Also, I have to find a non-negative function $g(n)$ such that for all the above function $f_i(n)$, $g(n)$ is nor $O(f_i(n))$ neither $\Omega (f_i(n))$.

Could you give me some hints how I could find such a function?? (Wondering)

So g is not bounded above by any of the functions, nor is it bounded from below. (Thinking)

That means that g keeps going up above the highest function, and keeps going below the lowest function.

Which function would be above all others?
Which function would be below all others?
Which function goes up and down? (Wondering)
 
  • #8
I like Serena said:
Looks good to me. (Nod)

I selected the first 6 and put them in this plot.
It confirms that those are in the correct order. (Happy)

For a justification, I think you only need to give the limits that prove that your order is the correct one. No need to do so many more. It would be nice if some of the actual calculation of the non-trivial ones were shown. (Nerd)

Ok... Nice! (Sun)
I like Serena said:
So g is not bounded above by any of the functions, nor is it bounded from below. (Thinking)

That means that g keeps going up above the highest function, and keeps going below the lowest function.

Which function would be above all others?
Which function would be below all others?
Which function goes up and down? (Wondering)

How can we find such a function?? (Wondering) I don't have any idea...
 
  • #9
mathmari said:
How can we find such a function?? (Wondering) I don't have any idea...

I think $2^{2^{2^n}}$ will go above all others, won't it? (Thinking)
 
  • #10
I like Serena said:
I think $2^{2^{2^n}}$ will go above all others, won't it? (Thinking)

Yes, but this function is bounded below, isn't it?? (Wondering)
We are looking for a function that isn't bounded below, don't we?? (Wondering)
 
  • #11
mathmari said:
Yes, but this function is bounded below, isn't it?? (Wondering)
We are looking for a function that isn't bounded below, don't we?? (Wondering)

That's why we have to tweak it to go up and down.
How could we do that?
Which functions do you know that go up and down? (Wondering)
 
  • #12
I like Serena said:
That's why we have to tweak it to go up and down.
How could we do that?
Which functions do you know that go up and down? (Wondering)

Do we have to multiply it with a trigonometric function?? (Wondering)
 
  • #13
mathmari said:
Do we have to multiply it with a trigonometric function?? (Wondering)

That is indeed one way to do it (sine wave).
Alternatively we can use a piece wise function (square wave).

Suppose we have functions $p(n)$ and $q(n)$ that are respectively above and below all functions.
Then $(1+\sin 2\pi n)p(n) + (1-\sin 2\pi n)q(n)$ should do the job.

Alternatively, we can pick:
\begin{cases}p(n) & \text{ if $n$ odd} \\ q(n) & \text{ if $n$ even}\end{cases}
(Nerd)
 
  • #14
I like Serena said:
Alternatively, we can pick:
\begin{cases}p(n) & \text{ if $n$ odd} \\ q(n) & \text{ if $n$ even}\end{cases}
(Nerd)

What functions could we take for example?? (Wondering)

The one of them $2^{2^{2^n}}$?? And the other one?? (Wondering)
 

1. What is the order of functions in scientific research?

The order of functions in scientific research typically follows these steps: problem identification, hypothesis development, data collection, data analysis, and conclusion.

2. How do you determine the order of functions in a scientific experiment?

The order of functions in a scientific experiment is determined by the nature of the research question and the type of data that needs to be collected. It is important to follow a logical sequence that will allow for accurate data collection and analysis.

3. Can the order of functions vary in different scientific disciplines?

Yes, the order of functions can vary in different scientific disciplines. For example, in social sciences, data collection often comes before hypothesis development, while in natural sciences, hypothesis development typically comes before data collection.

4. Is it possible to skip a step in the order of functions?

In some cases, it may be possible to skip a step in the order of functions, but this should be done with caution. Skipping a step can potentially lead to flawed results and conclusions. It is important to carefully consider the purpose and significance of each step before deciding to skip it.

5. How important is it to follow the order of functions in scientific research?

Following the order of functions in scientific research is crucial for ensuring the validity and accuracy of the results. Each step serves a specific purpose and skipping or rearranging them can compromise the integrity of the research. It is important to carefully plan and execute each step in the proper order.

Similar threads

  • Set Theory, Logic, Probability, Statistics
Replies
2
Views
1K
Replies
0
Views
369
  • Set Theory, Logic, Probability, Statistics
Replies
3
Views
2K
  • Set Theory, Logic, Probability, Statistics
Replies
1
Views
927
  • Set Theory, Logic, Probability, Statistics
Replies
1
Views
2K
  • Set Theory, Logic, Probability, Statistics
Replies
8
Views
1K
  • Set Theory, Logic, Probability, Statistics
Replies
1
Views
750
  • Set Theory, Logic, Probability, Statistics
Replies
1
Views
869
  • Set Theory, Logic, Probability, Statistics
Replies
2
Views
1K
  • Set Theory, Logic, Probability, Statistics
Replies
3
Views
2K
Back
Top